Saturday, April 27, 2024

2024/032) What is the proof that 11 is the only prime number of the form $n^2 + 2$ where n is prime?

We have a prime number is 2 or 3 of of the form $6n\pm  1$ for $n \gt 0$ let us compute $n^2+2$

$2^2 + 2 = 6 = 2 * 3$ not a prime

$3^2+ 2  = 11$ is a prime

$(6n\pm  1)^2+ 2 = (36n^2 \pm 12 n + 1) + 2 =  (36n^2 \pm 12 n + 3)  = 3(12n^2 \pm 4 n + 1)$ which is not a prime

hence 11 is the only prime

2024/031) Find triangular number which is one less than a multiple of 11.

 We have $n^{th}$ triangular number  $t_n= \sum_{k=1}^n k = \frac{n(n+1)}{2}$

 So we must have  $\frac{n(n+1)}{2} \equiv -1 \pmod {11}$

Or $n(n +1) \equiv -2 \pmod {11}$

Or $n^2 + n + 2 \equiv 0 \pmod {11}$

Or $4 n^2 + 4n + 8 \equiv 0 \pmod {11}$ (the purpose of doing this is to covert to perfect square as evident from next line)

Or $(2n+1)^2 + 7 \equiv 0 \pmod {11}$

Or  $(2n+1)^2 =  \equiv -7 \pmod {11}$

Or  $(2n+1)^2 =  \equiv 4 \pmod {11}$

Let us find the square mod 11 for n = 0 to 5 we get (0,0),(1,1),(2,4),(3,9),(4,5),(5,3)$

So the numbers are 2 and 9 ( that is 11 -2)

$2n + 1 \equiv  2  \pmod {11}$ gives n = 6 and $2n + 1 \equiv  9  \pmod {11}$ gives n = 4

So we have the triangular numbers are $t_{11k+4}$ and $t_{11k+6}$ for any non negative k

Sunday, April 21, 2024

2024/030) Prove that for triangular numbers $t_{3k}+t_{4k+1}=t_{5k+1}$

We have by definition  triangular number $t_k = \frac{k(k+1)}{2}$

So   $t_{3k}+t_{4k+1}$

$= \frac{(3k(3k+1)}{2} + \frac{(4k+1)(4k+2)}{2}$

$= \frac{(9k^2+3k}{2} + \frac{16k^2 + 12 k + 2}{2}$

$= \frac{25k^2+15k + 2}{2}$

$= \frac{(5k+1)(5k + 2)}{2}= t_{5k+1} $

Saturday, April 13, 2024

2024/029) Consider the equation $x^{2021} + x^{2020} + \cdots+x - 1=0$

 Then

a) exactly one real root is -ve

b) all real roots are positive

c) exactly one real root is positive

d) no real root is positive.

Solution

we have 

 $x^{2021} + x^{2020} + \cdots+x - 1=0$

or   $x^{2021} + x^{2020} + \cdots+x + 1=2$

Note that 1 is not a root of this equation.

 Multiplying both sides by $x-1$ we get

$x^{2022} - 1  = 2(x-1)$

or  $f(x) = x^{2022} - 2x  + 1 = 0$

 Note that as we have multiplied by x-1 so x = 1 so there is at least one positive root that is 1

As there is change of sign two times as per Descarte rule there are two or zero positive roots root but as it has at least one positive root so there are two positive roots so original equation has one positive root.

Now  $f(-x) = x^{2022} + 2x  + 1 = 0$

As there is no change of sign so there is no -ve root

 So the equation has has one root and it is positive.

So answer is (c)-



Descartes

Thursday, April 11, 2024

2024/028)What is the next term in series 8, 15, 24, and 35 and find the general term

 We have the terms 8,15,24,35

Let us compute 1st order difference 7,9,11 which is an AP

Next difference is 13 and so  next term is 35 + 13 = 48

So the nth term of the given sequence is quadratic say $an^2+bn + c$

Putting n =1 ,n= 2 and n =3 we get following 

$a+b + c = 8\cdots(1)$

$4a+2b + c = 15\cdots(2)$

$9a+3b + c = 24\cdots(3)$

Subtracting (1) from (2) we get

$3a+b = 7\cdots(4)$

Subtracting (2) from (3) we get

$5a+b = 9\cdots(5)$

Subtracting (4) from (3) we get $2a=2$ or $a=1$

Putting $a=1$ in (4) we get $b=4$ and putting in (1) we get $c=3$

So $n^{th}$ term = $n^2+4n + 3$ 

we get the same results by putting n=1,2,3,4 so on 

 

 

Tuesday, April 9, 2024

2024/027) A number n has sum of digits 100 while 44n has sum of digits 800 Find the sum of digits of 3n


The number can contain only the digits 1,2 besides 0. 1 * 44 = 44 and there is no overflow( if in the number the sum is one then it becomes 8, and if it is 2 the the sum of digits is 16) so the sum of digits is 8 times. if any digit is 3 to 9 then sum of digits less than 8 times so this shall give a lesser sum

(just an observation and not required for the result) Again 1 may be preceded/succeeded  by 0 or 1 as 11 * 44 = 484 . but 2 has to be    preceded/succeeded  by 0 as 21 * 44 = 924 and 12 * 44 = 538 and the sum of digits become less

So the number shall have 0 1 and 2 meeting above conditions so that sum of digits 100 and when we multiply by 3 (that is 3n) the digits shall be 0,3,6 and there is no overflow and sum of digits 300.

2024/026) What is the largest number of consecutive positive integers whose sum is exactly 2024?

 Let the number of numbers be n and it starts at a

So we have the sum $= an + frac{n(n-1)}{2} = 2024$

Or $2an + n(n-1) = 4048$ and $a\ge 1$

Or $n(2a+n-1)=  4048 = 16 * 253 = 2^4  * 23 *11$

Now out of n and 2a+n-1 one is  even and one is odd so n = 16 ( 2a +n -1 = 253) or 1 ( 2a +n -1 = 4048)  or 11 ( 2a +n -1 = 368) or 23 ( 2a +n -1 = 176)

Clearly n = 23 is the largest giving 2a + 22 = 176 and a = 77

So   largest number of consecutive positive integers is 23 and it starts with 77


Sunday, April 7, 2024

2024/025)What is the third number if the two numbers are 48 and 60? The L.C.M is 1680, and the GCD is 12 of all the numbers.

 LCM of 48 and 60 (that is 12 * 4 and 12 *5) = 240

Now LCM 1680 = 240 * 7

So 3rd number should have a factor 7 and it should be multiple of 12(else GCD cannot be be 12) and can have other that is 3rd factor that is a factor of 240/12 or 20 it could be 1,2,4,5,10,20. as GCD of 48 and 60 is 12 so taking a higher multiple shall not change GCD

So 3rd number could be 84/168/324/420/840/1680 that is any of these 6 numbers

Friday, April 5, 2024

2024/024) If $\cos\, A +\cos\, B + \cos\, C = 0$, prove that $\cos 3A + \cos 3B +\cos 3C = 12 \cos\, A \cos\, B \cos\, C$.

 We shall use 2 facts

$\cos 3x = 4 \cos^3 x - 3 \cos\, x\cdots(1)$

And 

if $a+b+c=0$ then $a^3+b^3+c^3 = 3abc\cdots(2)$

We are given

 $\cos\, A +\ cos\, B + \cos\, C = 0\cdots(3)$

Hence $\cos^3 A +\ cos^3 B + \cos^3 C = 3 \cos\, A \cos\, B \cos\, C\cdots(4)$

Now $\cos 3A + \cos 3B +\cos 3C$

$ = 4 \cos^3 A - 3 \cos\, A$ + $ 4 \cos^3 B - 3 \cos\, B $ + $  4 \cos^3 C - 3 \cos\, C$ using(1)

$=4(\cos^3 A +\ cos^3 B + \cos^3 C - 3 (\cos\, A + \cos\, B + \cos\, C)$

$ =4 * 3 \cos\, A \cos\, B \cos\, C- 3 *0 $  using (3) and (4)

$ =12 \cos\, A \cos\, B \cos\, C$

Proved


Thursday, April 4, 2024

2024/023) If a, b, c be in Arithmetic Progression, then the value of $(a+2b-c)(c+2b-a)(a+2b+c)$ is nabc. Find n

 We have a,b,c are in AP so 

$2b = a+ c$

Hence $a+2b -c = a + a+c -c = 2a\cdots(1)$

$c + 2b -a = c + a + c - a = 2c\cdots(2)$

$a + 2b + c= 2b + a +c = 2b+2b = 4b\cdots(3)$ 

Multiplying (1) (2) and (3) we get $(a+2b-c)(c+2b-a)(a+2b+c)= 16abc$

Hence n = 16

 

2024/022) N is a 50 digit no. all digit except the 26^th (from left) are 1.if n is divisible by 13,find the 26^th digit?

Let the $26^{th}$ digit be k

so $N = 1111\cdots( 50\, times) + (k-1) * 10^{24}$

or $N = \frac{10^{50}-1}{9} + (k-1) * 10^{24}$

or     $9N = 10^{50}-1 + 9 *  ((k-1) * 10^{24})$

because $GCD(9,13)=1$ so if n is divisible by 13 9N is divisible by 13.

as 13 ia prime we have

$10^{12} \equiv 1 \pmod {13}$

so  

$10{24} \equiv 1 \pmod {13}\cdots(1)$

 and

$10{48} \equiv 1 \pmod {13}$

hence

 $10{50} \equiv 100 \pmod {13}$

 or

 $10{50} \equiv 9 \pmod {13}$

$9N = 10^{50}-1 + 9 *  ((k-1) * 10^{24}) \pmod {13}$

$= 9-1  + 9 *  ((k-1) * 1) \pmod {13}$

 $= 9k -1 \pmod {13}$

this is zero and because k is a digit trying from 0 to 9 we get k=3 satisfies the condition

so the $26^{th}$ digit is 3 

Saturday, March 30, 2024

2024/021) Show that $\sin 3\theta = \sin \theta + 2 \sin \theta \cos 2\theta$ and use it to prove $\sin \frac{\pi}{9} + 2 \sin \frac{\pi}{9} \cos \frac{2\pi}{9} = \frac{\sqrt{3}}{2}$

LHS

$= \sin 3 \theta = \sin ( \theta + 2\theta) = \ sin \theta \cos 2 \theta + \cos \theta \sin 2\theta$ usng $\sin(A+B)$ formula 

$= \ sin \theta \cos 2 \theta + \cos \theta (2\sin \theta \cos \theta)$ using $\sin 2\theta$ formula

$= \ sin \theta (\cos 2 \theta + 2  \cos^2  \theta)$

$= \ sin \theta ( 2 \cos 2 \theta +  \cos 2  \theta + 1 )$ using formula for $\cos 2\theta$

$= \ sin \theta ( 2 \cos 2 \theta +   1 )$

$= 2 \ sin \theta  \cos 2 \theta +  \sin   \theta $ which is RHS

In the above putting $\theta = \frac{\pi}{3}$  we get

$\sin \frac{\pi}{9} +  2 \sin \frac{\pi}{9} \cos \frac{2\pi}{9} = \sin \frac{\pi}{3} = \frac{\sqrt{3}}{2}$

 

Sunday, March 24, 2024

2024/020) Show that there are infinitely many composite numbers of the form $10^n +3$ (n = 1, 2, 3, ... ).

If n = 2 we have $10^2+3 = 103$ this is divisible by 103 . 

Hence  $10^2 + 3 = 0 \pmod {103}\cdots(1)$

We know that 103 is a prime but we shall use this as a starting point to show that there are infinite numbers of the form $10^n+3$ is divisible by 103.

As 103 is a prime so using Fermats little theorem

$10^{102} = - 1 \pmod {103}$

Squaring both sides we get

$10^{204} =  1 \pmod {103}$

So from (1) and above we have

$10^{204n + 2} + 3=  0 \pmod {103}$

except for n = 0 when the value is 103 all other values are above 103 and have a factor 103 so not a prime

So there are infinitely many composite numbers as they are divisible by 103. We have not taken all cases and have found infinitely many composite numbers. Hence proved

 


2024/019) Given $a_n = 6^n + 8^n$ find the remainder when $a_{83}$ is divided by 49

 We shall use Euler's theorem that is a generalization of Fermat's little theorem: For any n and any integer a coprime to n, one has 

$a^{\phi(n)} \equiv -1 \pmod n$

Where $\phi(n)$ is number of numbers that is co-prime to n,

Let us compute $\phi(49)$ 

If $n = p_1^{k_1}p_2^{k_2}p_3^{k-3}  ... $ when each $p_i$ is  prime then

 $\phi(n)=n(1-\frac{1}{p_1})(1-\frac{1}{p_2})(1-\frac{1}{p_3})$

As $49= 7^2$ so $\phi(49) = 49 * (1-\frac{1}{7}) = 49 * \frac{6}{7} = 42$

Hence  

$6^{\phi(49)} \equiv -1 \pmod {49}n$

or $6^{(42)} \equiv -1 \pmod {49}$

Squaring both sides

 $6^{(84)} \equiv 1 \pmod {49}$

Dividing by 6 we get

  $6^{(83)} \equiv \frac{1}{6} \pmod {49}\cdots(1)$

Where $\frac{1}{6}} is not fraction but inverse of 6 mod 49

Similarly

  $8^{(83)} \equiv \frac{1}{8} \pmod {49}\cdots(2)$

 Adding (1) and(2) we get 

  $6^{(83)}+ 8^{(83)} \equiv \frac{1}{6}  + \frac{1}{8} \pmod {49}\cdots(2)$

Now $\frac{1}{6}  + \frac{1}{8} \pmod {49}$

$= \frac{8 + 6}{48} \pmod{49}$

$= \frac{14}{48} \pmod{49}$

$= \frac{14}{-1} \pmod{49}$ as $48 \equiv -1 \pmod {49}$

$= -14 \pmod{49}$

$=35$ taking positive number 

Hence remainder = 35 

Sunday, March 17, 2024

2024/018) There are 1000 coins -- 999 are fair, and 1 has heads on both sides. You randomly choose a coin and flip it 10 times. Miraculously, all 10 flips turn up heads. What is the probability that you chose the unfair coin

Probability that you choose one unfair coin = $\frac{1}{1000}$

iI you choose one unfair coin the probability that all 10 heads come = 1

So   Probability that you choose one unfair coin and all heads come = $\frac{1}{1000}$

Probability that you choose one fair coin = $\frac{999}{1000}$

If you choose one fair  coin the probability that all 10 heads come = $\frac{1}{2^{20}} = \frac{1}{1024}$

So   Probability that you choose one fair coin and all heads come = $\frac{999}{1024*1000}$

 So probability that all heads come = $\frac{999}{1024*1000} + \frac{1}{1000} = \frac{2023}{1024000}$

 So probability that coin is unfiar = $\frac{\frac{1}{1000}}{\frac{2023}{1024000}}=\frac{1024}{2023}$

 

Saturday, March 2, 2024

2024/017) integrate sin ax cos bx

We have $\sin \, ax + \cos \, bx = \frac{1}{2} (\sin (a+b) x + \sin (a-b) x$

Knowing that 

$\frac{d}{dx} \ cos \, nx = - n \sin \, nx$

Hence $\int  (\sin \, ax  \cos \, bx) dx  = \int (\frac{1}{2} (\sin (a+b) x + \sin (a-b) x)dx $ 

= $\frac{\cos(a+b)x}{a+b} - \frac{\cos(a-b)x}{a-b} + C $ 


Friday, February 23, 2024

2024/016) Prove that product of 4 consecutive positive integers cannot be a perfect cube.

Proof;

We  shall use the fact that if x and y are co-prime then xy is a perfect cube if both  x and y are perfect cubes.

Let the 1st number be x so the product is $x(x+1)(x+2)(x+3)$

Either x is odd or even

If x is odd then we have $GCD(x,x+2) = GCD(x+1, x+2) = GCD(x+2,x+3) = 1$

For x + 2 to be a perfect cube minimum x is 6 

So $GCD(x+2,x(+1)(x+2)) = 1$

So we need to prove that $x(x+1)(x+3)$ is not a perfect cube.

$x(x+1)(x+3) = x^3 + 4x^2 + 3$

We have $x(x+3) - (x-1)^2 = x^3+3x - x^2 -2x -1=  x-1 > 0$ as x is minimum 6 

So $x(x+1)(x+3) >= (x+1)^3$

  $x^3 + 4x^2 + 3$ is below  $(x+2)^3 = x^3 + 6x^2 + 12x + 8$

As it is between 2 cubes it cannot be a cube

Similarly we can prove taking $x +1$ when $x$ is even


 

Friday, February 16, 2024

2024/015) Find natual number n such that $2^n + n | 8^n + n $

 1,2,4,6 

We know that $ x+ y | x^3+y^3 $

Hence $2^n + n | (2^3)^n + n^3$

as $2^n + n | 8^n + n $

so $2^n + n | n^3-n  $

so we must have $ n^3-n =0 $ or $2^n + n <  n^3-n  $

$n^3-n= 0$ gives n = -1,0, 1 and out of theses only 1 is solution

we need to solve  $2^n + n <  n^3-n  $ or  $2^n  <  n^3-2n  $

let us find an upper bound for n putting a condition 

$2^n < n^3$ for  $n \lt 10$

putting n from 1 to 9 we see that $n \in \{1, 2,4,6\} $ satisfy the case and there is no other solution                                                                                                                                               

 

 

2024/014) Solve $x^2-y=111$ and $y^2-x=111$ for $ x\ne y$

 We are given 

 $x^2-y=111\cdots(1)$

 $y^2-x=111\cdots(2)$

From (1) and (2)

$x^2-y = y^2 -x$

Or $x^2-y^2 + x -y = 0$

Or $(x-y)(x+y) + (x-y) = 0$

Or  $(x-y)(x+y+1) = 0$

As  $ x\ne y$  dividing by x-y we get $x+y+1=0\cdots(3)$

Or $y = -(x+1)$

Putting the value in (1) we get $x^2 + (x+ 1) = 111$ or $x^2 + x - 110 = 0$

Or $(x-10)(x+11) = 0 $

Or $ x= 10$ or $x = -11$

Putting in (3) if $x= 10$ then $y = -11$

If $x= -11 $ then $y = 10$

So Solution set $x=10,y= -11$ or $x=-11,y=10$


Wednesday, February 14, 2024

2024/013) Prove that $123123 | 2^{60}-1$

To show that 123123 is a factor of $2^{60}-1$ we need to show that each prime factor of 123123 to the highest power is a factor

Now let us factor 123123

$123123= 123 * 1001 = 3 * 41 * 7 * 11 * 13$ 

As each prime occurs once we need to prove each of 3,7,11,13,41 divides  $2^{60}-1$

We know $x^y-1 | x^{my}-1$ for any m

We have $2^2-1 |  2^{60}-1$  and $3| 2^{2} -1$ using FLT(Fermat's Little Theorem)  so $3| 2{60}^-1$

$2^6-1 |  2^{60}-1$  and $7| 2^{6} -1$ using FLT(Fermat's Little Theorem)  so $7| 2{60}^-1$

$2^{10}-1 |  2^{60}-1$  and $11| 2^{10} -1$ using FLT(Fermat's Little Theorem)  so $11| 2{60}^-1$

 $2^{12}-1 |  2^{60}-1$  and $13| 2^{12} -1$ using FLT(Fermat's Little Theorem)  so $13| 2{60}^-1$

Now we need to prove for 41

using FLT we know  $41| 2^{40} -1$

So let us find $GCD(2^{(60}-1,2^{40}-1)$

$GCD(2^{(60}-1,2^{40}-1)= GCD((2^{(60}-1) - (2^{40-1},2^{40}-1)$

$=GCD((2^{60} - 2^{40}), 2^{40}-1)$

$=GCD((2^{40}( 2^{20}-1), 2^{40}-1)$

 $=GCD((2^{20}-1), 2^{40}-1)$ as second number is odd so removing even factors of 1st number

$= 2^{20}-1$ as this is a factor of  $ 2^{40}-1$

now $= 2^{20}-1)= (2^{10}-1)(2^{10}+1)$

$=  1023 * 1025$ as $41 | 1025$ so $41 | 2^{60}-1 $

as each of 3,7,11,13,41 divides  $2^{60}-1$ so 123123 is a factor

Friday, February 9, 2024

2024/012) Given $a+b+c$ is divisible by 6 prove that $a^3+b^3+c^3$ is divisible by 6

We have $a^3-a= a(a^2-1) = a(a-1)(a+1) = (a-1)a(a+1)$ 

As $(a-1)a(a+1)$ is product of 3 consecutive  if is s divisible by 6 say 6m for some m

So $a^3 = a + 6m\cdots(1)$

Similarly 

$b^3 = b + 6p\cdots(2)$

and

$c^3 = c + 6q\cdots(3)$

Adding (1),(2) and (3) we get $a^3+b^3+c^3 = (a+b+c) +6(m+p+q)$

So if   $(a+b+c)$ is divisible by 6 then $a^3+b^3+c^3$ is divisible by 6


2024/011) Find all values of n such that $6^n+1$ has all digits same.

We have $6^2=36$ that is it ends with 6.

So we have all the powers of 6 end with 6

So $6^n+1$ shall end with 7

So we need to find n such that all the digits of $6^n+1$ must have all digits 7 and let it be k 7's/

So $6^n+1 = \frac{7}{9} (10^k -1 )$

Or $9(6^n+1) = 7 (10^k -1 )$for

Or  $9 * 6^n + 16 = 7 * 10^k\cdots(1)$

We have $2 | 6$ so  $2^2 | 6^2 $ so if $n \ge  5$ then we have  

$9 * 6^n + 16 \equiv 16 \pmod {32} $ for $n \ge  5$

So   $9 * 6^n + 16 \equiv m  \pmod {32} $ where $m \le 16$ and not zero

For $k \ge 5$ $ 7 * 10^5 \equiv 0  \pmod {32} $

So $k \le 4$ 

So we need to check for n such that  $9 * 6^n + 16 \le = 70000$

Or $6^n \le  7776$

Or $6^n+1 \le 7777$

We calculate for n = 1 $6^1 + 1 = 7$ meets criteria

 n = 2 $6^2 + 1 = 37$ does not meet criteria

n = 3 $6^3 + 1 = 217$ does not meet criteria

n =4  $6^4 + 1 = 1297$ does not meet criteria

n = 5 $6^5 + 1 = 7777 $  meets criteria

Other value of n takes the value out of range

So $ n \in \{1,5\}$

 

 


 


 

 

 

Sunday, February 4, 2024

2024/010) Prove that $18 ! \equiv -1 \pmod {437} $

 We first factorize 437

$437  = 19 * 23$

Now let compute mod relative to19 and 23

As 19 is prime number as as per Wilson'sTheorem we have

  $18 ! \equiv -1 \pmod {19}\cdots(1) $

As 23 is prime number as as per Wilson'sTheorem we have

  $22 ! \equiv -1 \pmod {23} $

Now  

$22  \equiv -1 \pmod {23}\cdots(2) $

$21  \equiv -2 \pmod {23} \cdots(3)$

$20  \equiv -3 \pmod {23}\cdots(4) $

$19  \equiv -4 \pmod {23}\cdots(5) $

As $22! = 22 * 21 * 20 * 19 * 18! $

So $22 ! \equiv -1 \pmod {23} $

$\implies  22 * 21 * 20 *  19 *18 ! \equiv -1 \pmod {23} $

 $\implies  (-1) *(-2) * (-3) * (-4) *18 ! \equiv -1 \pmod {23} $

 $\implies  24 *18 ! \equiv -1 \pmod {23} $

 $\implies  24 *18 ! \equiv -1 \pmod {23} $

$\implies  1 *18 ! \equiv -1 \pmod {23} $

$\implies  18 ! \equiv -1 \pmod {23}\cdots(6) $

Using (1) and (3) we get 

  $18 ! \equiv -1 \pmod {437} $

Proved

Friday, February 2, 2024

2024/009) Find $\sqrt{\sqrt{9} - \sqrt{8}}$

We have $8 = 2 * 4 = 2 * 2^2$

so    $\sqrt{8} = 2 \sqrt{2}$

now $\sqrt{9} = 3$

so   $\sqrt{\sqrt{9} - \sqrt{8}}$

=  $\sqrt{3 - 2\sqrt{2}}$

this is of the form  $\sqrt{n - 2\sqrt{n-1}}$ when n = 3

so  $\sqrt{3 - 2\sqrt{2}} = \sqrt{2 + 1 - 2 * \sqrt{2} * 1}$

$= \sqrt{(\sqrt{2})^2 + 1^2 - 2 * \sqrt{2} * 1} $

$= \sqrt{2} - 1$                                                                                                                                                                                                                           

Wednesday, January 31, 2024

2024/008) Find minimum Value of $(a+7)^2+(b+2)^2$ with Constraint $(a-5)^2+(b-7)^2=4$

We have the constraint  $(a-5)^2+(b-7)^2=4$ this is set of points lying in a circle with centre (5,7) and radius 2.

We need to find the minimum of  square of the distance of (a,b) from (-7,-2) and this is minimum when  (a,b) lies in the line from (5,7) to (-7,-2)

Distance from (5,7) to (-7,2) = $\sqrt{(5+7)^2 + (7+2)^2}= \sqrt{12^2 + 9^2} = 15$

So distance from (a,b) to (-7,  -2) is $15-2 = 13$

Minimum Value of $(a+7)^2+(b+2)^2= 13^2 =169$

Saturday, January 27, 2024

2024/007) Find the value of k such that $(\frac{1}{x+k} + \frac{k}{x-k} + \frac{2k}{k^2-x^2})(| x-k | -k) = 0$ has exactly one non -ve root



Reference: https://www.physicsforums.com/threads/discovering-the-solution-problem-of-the-week-261-apr-23rd-2017.1037211/
Note that we cannot have or . Multiply both sides by , we get or As cannot be we have , this gives 2 values of , where or . Hence there is no solution to the problem.

Reference: https://www.physicsforums.com/threads/discovering-the-solution-problem-of-the-week-261-apr-23rd-2017.1037211/

Note that we cannot have $x=k$ or $x = -k$ 

So multiplying both sides by $x^2-k^2$

We get 

$((x-k) + k(x+k) + 2k) )(| x-k |  -k) = 0$

Or $(x+k)(k+1))(| x-k |  -k)=0$

As x cannot be -k  $(| x-k |  -k)=0$

So we get 2 values of x that is 2k or 0

Hence no solution


2024/006) Show that $10^{th}$ digit of a power of 3 is always even

The power of digit of 3 shall have 1 or 3 or 7 or 9 as the unit digit.

Let us see some power of 3 that is 1,3,27,81(after this the sequence in the 1st digit repeats.

So we have (20n + x) where x is 1 or 3 or 7 or 9.

When we multiply by 3 we get 60n + 3 or 60n + 9 or 60n + 21 or 60n + 27 that is 60n + 3 or 60n + 9 or 20(3n+1) + 7 or 20(3n+1) + 9

So tens digit is even

 

    

Friday, January 26, 2024

2024/005) For m > 1 show that if $2^{2m+1} > n^2$ then $2^{2m+1} \ge n^2+7$

Because $m > 1$ $2m + 1\ge 4$  So    $2^{2m+1}$ is divisible by 16.

  $2^{2m+1} $ is double of $(2^m)^2$ and is not a square.

Now we consider 2 cases

Let us consider when n is odd 

Then $n= (2k+1)$

So $n^2 = (2k+1)^2 = 4k^2 + 4k + 1 = 4k(k+1)+1$

Ss k or k+1 is even we $n^2 \equiv 1 \pmod 8$ 

So $2^{2m+1}$ is divisible by 8 and minimum number above $n^2$ which is divisible by 8 is $n^2 + 7$

So we have proved for the case n is odd.

Let us consider when n is even

If  $n^2$ is divisible by 8 then $n^2$ is divisible by 16 and as $2^{2m+1}$ is divisible by 16 and n^2 is divisible by 16 

So $2^{2m+1}-n^2 = 16k$ for some positive k

So $2^{2m+1}-n^2 \ge16$

Hence  $2^{2m+1}-n^2 \gt 7$ or $2^{2m+1} \gt n^2+ 7$

If $n^2$ is even and is not divisible by 8 then n is of the form 4k+2

$n^2= 16k^2 + 16k + 4$

Or $n^2 \equiv 4 \pmod {16}$ 

 As $2^{2m+1} = 0 \pmod {16}$

So $2^{2m+1} \ge n^2 + 12 $

Hence  $2^{2m+1} \gt n^2 + 7 $

 we have proved for all 3 cases hence done

 


 

  

Wednesday, January 17, 2024

2024/004) Find the smallest a and b such that $\frac{1}{640} = \frac{a}{10^b}$

We have $640= 128 * 5 =  2^7 * 5$

To make it a power of 10 we need to multiply by $5^6$ giving $10^7$

so $a=5^6$ and b = 7 are the smallest values


Monday, January 15, 2024

2024/003) Find $\sum_{n=1}^{\infty} \frac{n^2}{10^n}$

Here let $frac{1}{10} = x$

 We have for $|x|  \lt 1$

 $\sum_{n=0}^{\infty} x^n= \frac{1}{1-x}$

Differentiating both sides wrt x we get

$\sum_{n=0}^{\infty} nx^{n-1}= \frac{1}{(1-x)^2}$

Multiplying both sides by x we get 

 $\sum_{n=0}^{\infty} nx^n= \frac{x}{(1-x)^2}$

Differenting both sides wrt x we get

$\sum_{n=0}^{\infty} n^2x^{n-1}= \frac{d}{dx} \frac{x}{(1-x)^2}$

Usimg $\frac{d} ({dx} \frac{u}{v})= \frac{v\frac{du}{dx} - u \frac{dv}{dx}}{v^2}$

We get 

 $\sum_{n=0}^{\infty} n^2x^{n-10} =\frac{(1-x)^2 . 1 - x . (x-2)}{(1-x)^4} = \frac{x +1}{(1-x)^3}$

Multiply by x on both sides we get $\sum_{n=0}^{\infty} n^2x^n = \frac{x^2 +x}{(1-x)^3}$

Putting value of $x = \frac{1}{10}$ we get the result


Thursday, January 11, 2024

2024/002) If x = $2^{\frac{1}{3}} + 2^{-\frac{1}{3}}$ , How do you prove that $2x^3-6x = 5$

 Let $a =  2^{\frac{1}{3}}\cdots(1)$

So $a^3 = 2\cdots(2)$ 

And $\frac{1}{a} = 2^{-\frac{1}{3}}\cdots(3)$

And  

$x = a+ \frac{1}{a}\cdots(4)$

Cubing both sides $x^3 = a^3 + \frac{1}{a^3} + 3 * a * \frac{1}{a} ( a + \frac{1}{a})$

Or $x^3 = a^3 + \frac{1}{a^3} + 3 * ( a + \frac{1}{a})$

Or  $x^3 = 2 + \frac{1}{2} + 3 *x$ from (2), 3) and (4)

Or $x^3 =  \frac{5}{2} + 3 *x$

Or $2x^3 = 5 + 6x$

Or $2x^3-6x = 5$

Proved


Monday, January 1, 2024

2024/001) Given $\frac{x-a}{x-b} + \frac{x-b}{x-a} = \frac{a}{b} + \frac{b}{a}$

Let  $\frac{x-a}{x-b} = m$ and $\frac{a}{b} =n $ 

So we get   $ m + \frac{1}{m} = n + \frac{1}{n}$

Or $m^2n + n = n^2m + m$

Or $m^2n - n^2m = m - n$

Or $mn(m -n ) = m - n$

Or  m-n = 0 Or $mn = 1$

case 1

m = n gives $\frac{x-a}{x-b} = \frac{b}{a}$

Or $b(x-a) = x(a-b)$ or $bx-ab = ax-ab$ or $bx = ax$ or $x(a-b)=0$ or $x = 0$ 


Case 2)

mn =1 gives 

$\frac{x-a}{x-b}*\frac{a}{b}=1$

or $a(x-a)=b(x-b)$

Or $ax-a^2=bx-b^2$

Or $ax-bx  = a^2-b^2$

Or $x(a-b) = a2-b^2$

Or $x = a + b$